please help me with my math homework

Please Help Me With My Math Homework

Answers

Answer 1
I hope it helps :DD You can leave a comment if you do not understand :)
Please Help Me With My Math Homework
Answer 2

Answer:

108/43

Step-by-step explanation:

[tex](6\frac{2}{3}-\frac{2}{3}) : (\frac{8}{9}+1\frac{1}{2}) =[/tex]

[tex]= 6 : (\frac{16}{18}+\frac{3}{2}) =[/tex]

[tex]= 6 : (\frac{16}{18}+\frac{27}{18}) =[/tex]

[tex]= 6 : (\frac{43}{18}) =[/tex]

[tex]= 6 * \frac{18}{43} =[/tex]

[tex]= \frac{108}{43}[/tex]


Related Questions

Leah is writing an equivalent equation which solves for y. What should be her next step?

3 x + 4 y = 8. 3 x minus 3 x + 4 y = 8 minus 3 x. 4 y = 8 minus 3 x.

Add 4y to both sides of the equation.
Subtract 4y from both sides of the equation.
Multiply both sides of the equation by 4.
Divide both sides of the equation by 4

Answers

Answer:

y= 2

Step-by-step explanation:

What is the EQUATION of the line PERPENDICULAR to y=-2x+5 and passing through the point (0,2)?

Answers

[tex]▪▪▪▪▪▪▪▪▪▪▪▪▪  {\huge\mathfrak{Answer}}▪▪▪▪▪▪▪▪▪▪▪▪▪▪[/tex]

The required equation is ~

[tex] \boxed{ \sf{y = \frac{x}{2} + 2 }}[/tex]

[tex] \large \boxed{ \mathfrak{Step\:\: By\:\:Step\:\:Explanation}}[/tex]

Let's find the slope of given line ~

[tex]y = - 2x + 5[/tex]

comparing it with general slope - intercept form of line (y = mx + c) we get, m = -2 (that is slope of the line)

let the slope of the required line be n

And, now since the required line Is perpendicular to the given line. the product of their slopes is -1

that is ~

[tex] - 2 \times n = - 1[/tex]

[tex]n = \dfrac{ - 1}{ - 2} [/tex]

[tex]n = \dfrac{1}{2} [/tex]

slope of required line is ~ 1/2

now, let's use the point - slope form of line to find the equation of required (perpendicular) line (using point (0 , 2) ~

that is ~

[tex]y - y_1 = m(x - x_ 1)[/tex]

here, m = slope ~

[tex] y - 2 = \dfrac{1}{2} (x - 0)[/tex]

[tex]y - 2 = \dfrac{x}{2} [/tex]

[tex]y = \dfrac{x}{2} + 2[/tex]

I hope it helps ~

What is the value for x?
А=73
B=(6x + 4)
C=(8y-7)
X=?

Answers

Answer:

x = 5

Step-by-step explanation:

The base angles ,A and C are equal

A = C = 73

The sum of the angles of the triangle are 180

A+C+B =180

73+73 + 6x+4=180

6x+150=180

Subtract 150 from each side

6x+150-150=180-150

6x = 30

Divide by 6

6x/6 = 30/6

x = 5

What is 8 times larger than 3

Answers

8 x 3 = 24 for should be 24

Only because 8 times larger than three seems like a multiplication problem..

I hope this helps if it is correct!

Answer:

(8+1) x 3 = 27

Step-by-step explanation:

this is correct rather than 8 x 3 = 24 because both sides of 8x3=24 are equal rather than the sum being greater than the statement.

Use an equation to find the value of k so that the line passing through (−4,k) and (6,−7) has a slope of −15.

Answers

Answer:

k = 143

Step-by-step explanation:

Using the slope formula

(k- -7)/(-4-6) =-15

(k+7)/(-10) = -15

k+7 =-15*-10

k+7 = 150

Subtract 7

k+7-7 = 150-7

k = 143

Use a table to find the solution of each equation.

1. 0 = 4 = 2y

2. 12 = 6-3b

3. -48 = -9-13n


could someone please make a step-to-step explanation for me so I could understand this pretty please.

Answers

Step-by-step explanation:

1.If the number 1 is:0=4-2y

y=2

2.b=-2

3.n=3

Find the coordinates of the image of (- 3, 4) under the transformation T- 2,3
A. (1, - 1)
B.(- 1, 1)
C.(5, - 7)
D.(- 5, 7)

Answers

Answer: D, (- 5, 7)

Step-by-step explanation:

Since the transformation is - 2, + 3 we just need to subtract 2 from the x value and add 3 to the y value.

- 3 - 2 = - 5

4 + 3 = 7

- 5, 7

-
Tyrese says that 4(3x - 2) = 4x - 8.
Explain why his answer is not correct.

Answers

You have to multiply everything inside the brackets by 4, including 3x so (4*3)x = 12x

So the correct answer is 12x - 8

Help me this question is so hard i fried up my brain yesterday working on it for so long!!!!

Answers

Hello there! (:

The answer is 9.

3^4=3*3*3*3 (81)

3^2=9

81:9=9

So the answer is 9.

Hope it helps! If you have any question or query, feel free to ask! (:

~An excited gal

[tex]SparklingFlower[/tex]

ticket to florida from a centain city costs 249 does the airplane collect if 385 tickests are sold

Answers

Answer:

Yes.  $21, 165

Step-by-step explanation:

since 385 is greater than 249. as long as theres more than 249

249 x 85 = 21,165

Answer:

No because they only charge 249 unless they hack there bank account........anything is possible.

Step-by-step explanation:

Which is greater? 650% or 6.5

Answers

Answer:

The greater answer is 650%

Given the two triangles are similar, what is the value of s? (DO NOT round your answer.)

Answers

Answer:

the value of s is 7.5 ...............

Answer:

s=7.5

Step-by-step explanation:

[tex]\frac{s}{6}=\frac{5}{4}[/tex]

[tex]s=6(\frac{5}{4})=\frac{30}{4} =7.5[/tex]

I hope this help you

A taxi cab charges a fixed amount of $1.50 in addition to $0.75 per mile. If Jasmine has
$20 with her, what is the maximum number of miles she can ride in the cab? Explain
your reasoning.

Answers

The inequality would be 1.50 + .75x is less than or equal to 20. To solve this you would need to subtracts 1.50 from both sides, this would get you .75x is less than or equal to 18.5 next you would need to divide .75 on both sides ending your inequality with x is less than or equal to 24.6 repeating. To use this in the problem jasmine would be able to ride 24 miles with $20.

Answer:

Jasmine would be able to go 18 Miles .

Step-by-step explanation:

.75 cents per mile multiply that by 24 (Miles)

You get 18 ( 18 Dollars )  Add the FIXED Fee of $1.50

the with her $20 dollars she can go 24 miles and still have .50

Cents left over . Hope That Helps :D !

Subtract using the number line.
[tex] - 1 \frac{1}{3} - \frac{1}{6} [/tex]

Answers

Answer:

- 1 2/6 - 1/6

- 1 3/6

-1 1/2

Step-by-step explanation:

Step-by-step explanation:

[tex] = - 1 \frac{1}{3} - \frac{1}{6} [/tex]

[tex] = - 1 + ( - \frac{1}{3} - \frac{1}{6} )[/tex]

[tex] = - 1 + ( - \frac{2}{6} - \frac{1}{6} )[/tex]

[tex] = - 1 - \frac{3}{6} [/tex]

[tex] = - 1 \frac{1}{2} [/tex]

Option → B

A chef cooks 12 lbs of chicken for 36 people and 22 lbs of chicken for 66 people.

Answers

Answer:

One pound of chicken serves 3 people. Each person gets 1/3 pounds

12/36=1/3

22/66=1/3

Hope this helps.

soooooooo I need help lol​

Answers

Answer:

x = 1

Step-by-step explanation:

4 - (-6) = 10

10/5 = 2

2 = 2x

divide both sides by 2:

2/2 = x

x = 1

Have a nice day! :-)

and please tell me if anything is wrong.

Answer:

4-x = 10

10/5 is the fraction

10/5 = 2

2 = 2x

Which expression is undefined?

Answers

Answer:

D

Reasoning:

Anything that is divided by zero is undefined.

Triangle R S T is shown. Angle R S T is a right angle. The length of R S is 10, the length of R T is 26, and the length of S T is 24. Use the diagram and side lengths of triangle RST to determine the angles used for the trigonometric ratios. sin( ) = Twelve-thirteenths tan( ) = Five-twelfths

Answers

Answer:

sin(R) = 12/13tan(T) = 5/12

Step-by-step explanation:

The mnemonic SOH CAH TOA is intended to remind you of the relationship between sides of a right triangle and trig functions of the acute angles. It tells you ...

  Sin = Opposite/Hypotenuse

  sin(R) = 24/26 = 12/13

and ...

  Tan = Opposite/Adjacent

  tan(T) = 10/24 = 5/12

Answer: SIN = R and TAN = T

HELP ASAP PLEASE 10 POINTS
What is the equation in point slope form of the line that passes through the point ( 2 , -4 ) and has a slope of 4?

Answers

Answer:

y + 4 = 4(x - 2)

Step-by-step explanation:

Point slope form: y - y1 = m(x - x1)

Where m = slope and (x1 , x2) = any given point.

We are given that the equation has a slope of 4 and passes through the point (2,-4)

So m = 4 and (x1,y2) = (2,-4)

We then plug in these values into the point slope form equation to get the answer.

y - y1 = m(x - x1)

m = 4 and (x1,y1) = (2,-4)

plug in values

y - (-4) = 4(x-2)

remove parenthesis on "-4" and change sign because the two negative signs cancel out and change into a positive sign

We end up with y + 4 = 4(x-2)

Kendra rides her bike to school each day. It takes her 10 minutes to ride 30 blocks. What unit rate would express the speed of Kendra's bike ride? (4 points)

Group of answer choices

2 blocks per minute

10 blocks per minute

3 blocks per minute

4 blocks per minute

Answers

Answer:

3 blocks a minute

Step-by-step explanation:

so if she rides her bike 30 blocks in 10 minutes in means she rides at a pace of 3 blocks a minute

Evaluate - 4/8 ./. 8 2/3

Answers

Answer:

The answer is 13/3.

Step-by-step explanation:

8 2/3 is an improper fraction and is expressed as 26/3. CCF (copy, change, flip) is then applied to the division statement and 26/3 to becomes 4/8*26/3. The final answer is 104/24 which simples to 13/3.

For each value of W,determine whether it is a solution to -8+4w ≤16.

Answers

w is equal to -2

explanation: add 16 plus -8
then divide 4/ -8
then it equals -2

Answer:

9 = No

6 = Yes

-4 = Yes

-10 = Yes

Step-by-step explanation:

[tex]-8+4w\leq 16\\\\4w\leq 24\\\\w\leq 6[/tex]

Determine if the ordered pairs (1. - 2) and (-4,2) are solutions of the following linear inequality in two variables.
-X<-y
.
Is the ordered pair (1. - 2) a solution of the linear inequality?

Is the ordered pair (-4,2) a solution of the linear inequality?

Answers

Answer:

First is, second is not.

Step-by-step explanation:

Start plugging them!

[tex]-(1) < -(-2) \rightarrow -1 < 2 \\ \\- (-4) < -(2) \rightarrow 4 < -2[/tex]

First one is true, so the pair is a solution.

Second is false, (4 is obviously bigger!) so it's not a solution.

If you end up having to check multiple values, your best bet is to draw the solutions on a convenient plane and see where the points lay

which rotation about its center will carry a regular hexagon onto itself

Answers

There are 6 angles between neighbour vertices, they all are equal (because a hexagon is regular) and their sum is 360°. Thus each angle has a measure of 360°/6=60°. Each subsequent rotation by 60° also maps a hexagon onto itself.

Evaluate the expression.

2−[(2+10(−1)÷2)+1]

What is the value of the expression?

Answers

Answer:

4

Step-by-step explanation:

Review

To evaluate the expression, lets use BODMAS. Before we get to the solving part, lets revise BODMAS.

=> B in BODMAS is brackets. This means that whatever operations are inside a bracket, we must solve that first.

=> O in BODMAS is Orders. The O is used very rarely, there is a possibility that there is no powers, exponents.

=> D in BODMAS is Division. This means that whatever division is occurring in the expression must be done first (Unless if there is a bracket).

=> M in BODMAS is Multiplication. This means that whatever multiplication is occurring in the expression must be done next.

=> A in BODMAS is Addition. This means that whatever addition is occurring in the expression must be done then.

=> S in BODMAS is Subtraction. This means that whatever subtraction is occurring in the expression must be done last.

Solution:

Now, lets get to the point. We need to find the expression '2−[(2+10(−1)÷2)+1]'. Using the help of BODMAS, we can solve it.

Step 1: Simplify the innermost bracket.

2 − [( 2 + 10(−1)÷2)+1]

=> 2-[(2-10÷2)+1]

Note: Since in the innermost bracket there is no operation, we can open the innermost bracket and get along with the second inner bracket.

Step 2: Simplify the second inner bracket.

=> 2 - [(2 - 10 ÷ 2) + 1]

=> 2 - [(2 - 5) + 1]

=> 2 - [(-3) + 1]

=> 2 - [-3 + 1]

Step 3: Simplify the third inner bracket.

=> 2 - [-3 + 1]

=> 2 + 2

Step 4: Solve.

=>  2 + 2

=> 4

Final Answer.

Therefore, 4 is our answer after working on this problem.

Answer:

4

Step-by-step explanation:the answer is 4 because you have to 20 2-2+10 and that is ten. then minus one is 9 then 9/2=3 and then 3+1 equals 4

5 1/2= ? I need help plz

Answers

Answer:

If it's a mixed fraction, it is 11/2 but if it's multiplication, it's 2.5. I don't really know what you mean.

A framer's market sells apples for $0.75 per pound and oranges for $0.89 per pound

Answers

Answer:

160

Step-by-step explanation:

jbuyvcezxrctvubnjonjbvcrexwexcrvbnmnbvcrxewexrcvtbn

Let's look at the first description.



How many dominoes have at least one 6 on them and the sum of the two numbers is even?

Answers

Answer:

28 tiles

Step-by-step explanation:

find the zeros of the polynomial equation x⁴+2x³-13x²-14x+24=0​

Answers

Answer:×¹ = -4, ײ = -2, ׳ = 1, ×⁴ = 3

Mark me as a Brainliest

Step-by-step explanation:

Hope this helps :)

i need help on this equation.​

Answers

Answer:

x

Step-by-step explanation:

Could you show me the drop boxes?
Other Questions
Which kind of psychology seeks to identify and promote those qualities that lead to happy, fulfilled, and contented lives?. How would you gather data to in order to determine the density of a marble Solve the following inequality Data gathered on the shopping patterns during the months of April and May of high school students from Peanut Village revealed the following. 38% of students purchased a new pair of shorts (call this event H), 15% of students purchased a new pair of sunglasses (call this event G) and 6% of students purchased both a pair of short and a pair of sunglasses. Which of the following CANNOT be prevented? any five problems related to employ ment in nepal If 2m +1= 3m - 3, what is the value of m? What is your walking rate in steps per minute? what pattern do you noticed for each column What should you not do during a hostage rescue attempt? A skydiver jumps from an airplane that is moving at 50 meters per second at a height of 1,000 meters what describes the skydivers energy immediately after leaving the air plane You randomly select one card from a 52-card deck. Find the probability of selecting a black four or a black five. For the function: f(x) = 3x^2 4x 8, find and simplifyA. f(x) - f(a) divided by x-ab.f(x +h) - f(x) all divided by H The table shows the portions of copperRingPortion that is CopperRed gold25%Pink gold0.2Rose gold940Which ring has the highest portion of copper? What do you know about world war 2 A mouse runs 2 metres in 4 seconds. What is its speed? 10 +4m when m = -2i really need help on this lol Compare your response to the sample response above. What did you include in your explanation?a reference to the horizontal-line testa statement that the function is one-to-onethe conclusion that the inverse is a function The 4th grade class met their goal of reading of certain amount of hours during the fall. They earned their skiing trip this winter sponsored by the parent-Teacher organization. if each ski ticket costs $28 and there are 86 fourth grades, how much does PTO need to pay for the tickets? How are the General and Rainsford alike?